"전자기학의 라그랑지안"의 두 판 사이의 차이

수학노트
둘러보기로 가기 검색하러 가기
 
(같은 사용자의 중간 판 4개는 보이지 않습니다)
1번째 줄: 1번째 줄:
 
==하전입자에 대한 라그랑지안==
 
==하전입자에 대한 라그랑지안==
* 전기장 $\mathbf{E}=\nabla \phi$
+
* 전기장 <math>\mathbf{E}=-\nabla \phi</math>
* 자기장 $\mathbf{B}=\nabla \times \mathbf{A}$
+
* 자기장 <math>\mathbf{B}=\nabla \times \mathbf{A}</math>
* 전자기장 안에 놓인 질량 $m$, 전하 $e$의 하전입자에 대한 라그랑지안
+
* 전자기장 안에 놓인 질량 <math>m</math>, 전하 <math>e</math>의 하전입자에 대한 라그랑지안
 
:<math>L(q,\dot{q})=\frac{m||\dot{q}||^2}{2}-e\phi+eA_{i}\dot{q}^{i}</math>
 
:<math>L(q,\dot{q})=\frac{m||\dot{q}||^2}{2}-e\phi+eA_{i}\dot{q}^{i}</math>
 
* 켤레운동량
 
* 켤레운동량
 
:<math>p_{i}=\frac{\partial{L}}{\partial{\dot{q}^{i}}}=m \dot{q}_{i}+eA_{i}=mv_{i}+eA_{i}</math>
 
:<math>p_{i}=\frac{\partial{L}}{\partial{\dot{q}^{i}}}=m \dot{q}_{i}+eA_{i}=mv_{i}+eA_{i}</math>
 
* [[오일러-라그랑지 방정식]] <math>\dot{p}=F</math>은 다음과 같이 쓰여진다
 
* [[오일러-라그랑지 방정식]] <math>\dot{p}=F</math>은 다음과 같이 쓰여진다
$$
+
:<math>
 
\dot{p}_{i}=m\frac{dv_{i}}{dt}+e\frac{\partial{A_{i}}}{\partial t}+e\frac{\partial{A_{i}}}{\partial{q}^{j}}\dot{q}^{j}=m\frac{dv_{i}}{dt}+e\frac{\partial{A_{i}}}{\partial{q}^{j}}\dot{q}^{j} \\
 
\dot{p}_{i}=m\frac{dv_{i}}{dt}+e\frac{\partial{A_{i}}}{\partial t}+e\frac{\partial{A_{i}}}{\partial{q}^{j}}\dot{q}^{j}=m\frac{dv_{i}}{dt}+e\frac{\partial{A_{i}}}{\partial{q}^{j}}\dot{q}^{j} \\
 
F_{i}=\frac{\partial{L}}{\partial{q^{i}}}=\frac{\partial}{\partial{{q}^{i}}}(-e\phi+eA_{j}\dot{q}^{j})=-e\frac{\partial{\phi}}{\partial{q}^{i}} +e\frac{\partial{A_{j}}}{\partial{q}^{i}}\dot{q}^{j}
 
F_{i}=\frac{\partial{L}}{\partial{q^{i}}}=\frac{\partial}{\partial{{q}^{i}}}(-e\phi+eA_{j}\dot{q}^{j})=-e\frac{\partial{\phi}}{\partial{q}^{i}} +e\frac{\partial{A_{j}}}{\partial{q}^{i}}\dot{q}^{j}
$$
+
</math>
 
:<math>m\frac{dv_{i}}{dt}=eE_{i}+eF_{ij}\dot{q}^{j},\quad i=1,2,3 \label{eom}</math>
 
:<math>m\frac{dv_{i}}{dt}=eE_{i}+eF_{ij}\dot{q}^{j},\quad i=1,2,3 \label{eom}</math>
여기서 $F_{\mu\nu} = \partial_\mu A_\nu - \partial_\nu A_\mu \,\!$
+
여기서 <math>F_{\mu\nu} = \partial_\mu A_\nu - \partial_\nu A_\mu \,\!</math>
* $F_{12}=B_{3}$, $F_{23}=B_{1}$, $F_{31}=B_{2}$
+
* <math>F_{12}=B_{3}</math>, <math>F_{23}=B_{1}</math>, <math>F_{31}=B_{2}</math>
* 가령 $i=1$이면, \ref{eom}은 다음과 같다
+
* 가령 <math>i=1</math>이면, \ref{eom}은 다음과 같다
$$
+
:<math>
 
ma_1=eE_1+e(F_{11}\dot{q}^{1}+F_{12}\dot{q}^{2}+F_{13}\dot{q}^{3})=eE_1+e(F_{12}\dot{q}^{2}-F_{31}\dot{q}^{3})=eE_1+e(\mathbf{v}\times \mathbf{B})_{1}
 
ma_1=eE_1+e(F_{11}\dot{q}^{1}+F_{12}\dot{q}^{2}+F_{13}\dot{q}^{3})=eE_1+e(F_{12}\dot{q}^{2}-F_{31}\dot{q}^{3})=eE_1+e(\mathbf{v}\times \mathbf{B})_{1}
$$
+
</math>
* 전하가 받는 힘 $\mathbf{F}$는 다음과 같다
+
* 전하가 받는 힘 <math>\mathbf{F}</math>는 다음과 같다
 
:<math>\mathbf{F}=e(\mathbf{E}+\mathbf{v}\times \mathbf{B})</math>
 
:<math>\mathbf{F}=e(\mathbf{E}+\mathbf{v}\times \mathbf{B})</math>
 
* 이를 로렌츠 힘이라 한다
 
* 이를 로렌츠 힘이라 한다
26번째 줄: 26번째 줄:
 
==전자기장에 대한 라그랑지안==
 
==전자기장에 대한 라그랑지안==
 
===상호작용이 없는 경우===
 
===상호작용이 없는 경우===
* $j$$\rho$가 0인 경우
+
* <math>j</math><math>\rho</math>가 0인 경우
 
* 전자기장에 대한 라그랑지안은 다음과 같다
 
* 전자기장에 대한 라그랑지안은 다음과 같다
$$\mathcal{L}_{\text{EM}}= - \frac{1}{4}F_{\mu\nu}F^{\mu\nu}=\frac{1}{2}(\mathbf{E}^2-\mathbf{B}^2)$$
+
:<math>\mathcal{L}_{\text{EM}}= - \frac{1}{4}F_{\mu\nu}F^{\mu\nu}=\frac{1}{2}(\mathbf{E}^2-\mathbf{B}^2)</math>
이 때 <math>F_{\mu\nu} = \partial_\mu A_\nu - \partial_\nu A_\mu \,\!</math>는 전자기텐서, $A=(A_{\mu})$는 전자기 포텐셜
+
이 때 <math>F_{\mu\nu} = \partial_\mu A_\nu - \partial_\nu A_\mu \,\!</math>는 전자기텐서, <math>A=(A_{\mu})</math>는 전자기 포텐셜
 
* 작용
 
* 작용
 
:<math>S=-\frac{1}{4}\int F^{\alpha\beta}F_{\alpha\beta}\,d^{4}x</math>
 
:<math>S=-\frac{1}{4}\int F^{\alpha\beta}F_{\alpha\beta}\,d^{4}x</math>
 
* 라그랑지안은 전자기 포텐셜의 다음과 같은 변환에 대하여 불변이다
 
* 라그랑지안은 전자기 포텐셜의 다음과 같은 변환에 대하여 불변이다
 
:<math>A_{\mu}(x) \to A_{\mu}(x)-\partial_{\mu}\Lambda(x)</math>
 
:<math>A_{\mu}(x) \to A_{\mu}(x)-\partial_{\mu}\Lambda(x)</math>
여기서 $\Lambda(x)$는 임의의 스칼라장
+
여기서 <math>\Lambda(x)</math>는 임의의 스칼라장
 
* 운동방정식
 
* 운동방정식
$$
+
:<math>
 
\partial_\mu F^{\mu\nu}=0
 
\partial_\mu F^{\mu\nu}=0
$$
+
</math>
  
 
===상호작용이 있는 경우===
 
===상호작용이 있는 경우===
* $j$$\rho$가 0이 아닌 경우
+
* <math>j</math><math>\rho</math>가 0이 아닌 경우
 
* 전자기장에 대한 라그랑지안은 다음과 같다
 
* 전자기장에 대한 라그랑지안은 다음과 같다
$$L=-\frac{1}{4}F_{\mu\nu}F^{\mu\nu}-ej_\mu A^\mu$$
+
:<math>L=-\frac{1}{4}F_{\mu\nu}F^{\mu\nu}-ej_\mu A^\mu</math>
 
* 작용
 
* 작용
$$S[\phi,A]=\int_{t_1}^{t_2}\int_{\mathbb{R}^3}\left(-\rho\phi+j\cdot A+\frac{\epsilon_0}{2}E^2-\frac{1}{2\mu_0}B^2\right)\,dV\,dt$$
+
:<math>S[\phi,A]=\int_{t_1}^{t_2}\int_{\mathbb{R}^3}\left(-\rho\phi+j\cdot A+\frac{\epsilon_0}{2}E^2-\frac{1}{2\mu_0}B^2\right)\,dV\,dt</math>
 
* 운동방정식
 
* 운동방정식
$$
+
:<math>
 
\nabla\cdot E=\frac{\rho}{\epsilon_0}\\
 
\nabla\cdot E=\frac{\rho}{\epsilon_0}\\
 
\nabla\times B=\mu_0j+\epsilon_0\mu_0\frac{\partial E}{\partial t}
 
\nabla\times B=\mu_0j+\epsilon_0\mu_0\frac{\partial E}{\partial t}
$$
+
</math>
  
  
75번째 줄: 75번째 줄:
 
* THOMAS YU [http://math.uchicago.edu/~may/REU2012/REUPapers/Yu.pdf Lagrangian formulation of the electromagnetic field]
 
* THOMAS YU [http://math.uchicago.edu/~may/REU2012/REUPapers/Yu.pdf Lagrangian formulation of the electromagnetic field]
 
* Lea, [http://www.physics.sfsu.edu/~lea/courses/grad/fldlagr.PDF The field Lagrangian]
 
* Lea, [http://www.physics.sfsu.edu/~lea/courses/grad/fldlagr.PDF The field Lagrangian]
* Susskind, [http://www.lecture-notes.co.uk/susskind/classical-mechanics/lecture-8/the-electromagnetic-lagrangian/ The electromagnetic Lagrangian]
 
 
* Lecture 8 | Modern Physics: Classical Mechanics (Stanford). 2008. http://www.youtube.com/watch?v=gUUbl444r74&feature=youtube_gdata_player.
 
* Lecture 8 | Modern Physics: Classical Mechanics (Stanford). 2008. http://www.youtube.com/watch?v=gUUbl444r74&feature=youtube_gdata_player.
 +
** Susskind, [http://www.lecture-notes.co.uk/susskind/classical-mechanics/lecture-8/the-electromagnetic-lagrangian/ The electromagnetic Lagrangian]
 +
* Special Relativity | Lecture 9. 2012. http://www.youtube.com/watch?v=sGyXdCb0l50&feature=youtube_gdata_player.
 +
** Susskind, [http://theoreticalminimum.com/courses/special-relativity-and-classical-field-theory/2012/spring/lecture-9 Lagrangian for Maxwell's Equations]
 
* http://www.damtp.cam.ac.uk/user/tong/qft/six.pdf
 
* http://www.damtp.cam.ac.uk/user/tong/qft/six.pdf
  
  
 
[[분류:수리물리학]]
 
[[분류:수리물리학]]
 +
 +
==메타데이터==
 +
===위키데이터===
 +
* ID :  [https://www.wikidata.org/wiki/Q172137 Q172137]
 +
===Spacy 패턴 목록===
 +
* [{'LOWER': 'lorentz'}, {'LEMMA': 'force'}]
 +
* [{'LOWER': 'electromagnetic'}, {'LEMMA': 'force'}]
 +
* [{'LOWER': 'coulomb'}, {'OP': '*'}, {'LOWER': 'lorentz'}, {'LEMMA': 'force'}]

2021년 2월 17일 (수) 03:23 기준 최신판

하전입자에 대한 라그랑지안

  • 전기장 \(\mathbf{E}=-\nabla \phi\)
  • 자기장 \(\mathbf{B}=\nabla \times \mathbf{A}\)
  • 전자기장 안에 놓인 질량 \(m\), 전하 \(e\)의 하전입자에 대한 라그랑지안

\[L(q,\dot{q})=\frac{m||\dot{q}||^2}{2}-e\phi+eA_{i}\dot{q}^{i}\]

  • 켤레운동량

\[p_{i}=\frac{\partial{L}}{\partial{\dot{q}^{i}}}=m \dot{q}_{i}+eA_{i}=mv_{i}+eA_{i}\]

\[ \dot{p}_{i}=m\frac{dv_{i}}{dt}+e\frac{\partial{A_{i}}}{\partial t}+e\frac{\partial{A_{i}}}{\partial{q}^{j}}\dot{q}^{j}=m\frac{dv_{i}}{dt}+e\frac{\partial{A_{i}}}{\partial{q}^{j}}\dot{q}^{j} \\ F_{i}=\frac{\partial{L}}{\partial{q^{i}}}=\frac{\partial}{\partial{{q}^{i}}}(-e\phi+eA_{j}\dot{q}^{j})=-e\frac{\partial{\phi}}{\partial{q}^{i}} +e\frac{\partial{A_{j}}}{\partial{q}^{i}}\dot{q}^{j} \] \[m\frac{dv_{i}}{dt}=eE_{i}+eF_{ij}\dot{q}^{j},\quad i=1,2,3 \label{eom}\] 여기서 \(F_{\mu\nu} = \partial_\mu A_\nu - \partial_\nu A_\mu \,\!\)

  • \(F_{12}=B_{3}\), \(F_{23}=B_{1}\), \(F_{31}=B_{2}\)
  • 가령 \(i=1\)이면, \ref{eom}은 다음과 같다

\[ ma_1=eE_1+e(F_{11}\dot{q}^{1}+F_{12}\dot{q}^{2}+F_{13}\dot{q}^{3})=eE_1+e(F_{12}\dot{q}^{2}-F_{31}\dot{q}^{3})=eE_1+e(\mathbf{v}\times \mathbf{B})_{1} \]

  • 전하가 받는 힘 \(\mathbf{F}\)는 다음과 같다

\[\mathbf{F}=e(\mathbf{E}+\mathbf{v}\times \mathbf{B})\]


전자기장에 대한 라그랑지안

상호작용이 없는 경우

  • \(j\)와 \(\rho\)가 0인 경우
  • 전자기장에 대한 라그랑지안은 다음과 같다

\[\mathcal{L}_{\text{EM}}= - \frac{1}{4}F_{\mu\nu}F^{\mu\nu}=\frac{1}{2}(\mathbf{E}^2-\mathbf{B}^2)\] 이 때 \(F_{\mu\nu} = \partial_\mu A_\nu - \partial_\nu A_\mu \,\!\)는 전자기텐서, \(A=(A_{\mu})\)는 전자기 포텐셜

  • 작용

\[S=-\frac{1}{4}\int F^{\alpha\beta}F_{\alpha\beta}\,d^{4}x\]

  • 라그랑지안은 전자기 포텐셜의 다음과 같은 변환에 대하여 불변이다

\[A_{\mu}(x) \to A_{\mu}(x)-\partial_{\mu}\Lambda(x)\] 여기서 \(\Lambda(x)\)는 임의의 스칼라장

  • 운동방정식

\[ \partial_\mu F^{\mu\nu}=0 \]

상호작용이 있는 경우

  • \(j\)와 \(\rho\)가 0이 아닌 경우
  • 전자기장에 대한 라그랑지안은 다음과 같다

\[L=-\frac{1}{4}F_{\mu\nu}F^{\mu\nu}-ej_\mu A^\mu\]

  • 작용

\[S[\phi,A]=\int_{t_1}^{t_2}\int_{\mathbb{R}^3}\left(-\rho\phi+j\cdot A+\frac{\epsilon_0}{2}E^2-\frac{1}{2\mu_0}B^2\right)\,dV\,dt\]

  • 운동방정식

\[ \nabla\cdot E=\frac{\rho}{\epsilon_0}\\ \nabla\times B=\mu_0j+\epsilon_0\mu_0\frac{\partial E}{\partial t} \]


메모


관련된 항목들


매스매티카 파일 및 계산 리소스


사전 형태의 참고자료


리뷰, 에세이, 강의노트

메타데이터

위키데이터

Spacy 패턴 목록

  • [{'LOWER': 'lorentz'}, {'LEMMA': 'force'}]
  • [{'LOWER': 'electromagnetic'}, {'LEMMA': 'force'}]
  • [{'LOWER': 'coulomb'}, {'OP': '*'}, {'LOWER': 'lorentz'}, {'LEMMA': 'force'}]